LSAT and Law School Admissions Forum

Get expert LSAT preparation and law school admissions advice from PowerScore Test Preparation.

 kristinaroz93
  • Posts: 160
  • Joined: Jul 09, 2015
|
#21779
"A smokertrying to quit is more likely to succeed if his/her doctor greatly exaggerates the dangers of smoking..."

I selected A and I am having a hard time seeing why it is a worse answer than D.

Seeing as how this is an assumption question,we take what is in the answer choice, negate it, and then see whether or not it breaks down the argument.

Choice A negated states that people do not tend to believe outside sources -(i.e. doctors)

Since the argument states that people are more successful at quitting bad habits when the consequences are exaggerated to them and that this requires a degree of deception which needs to come from a third party, isn't it imperative for us to then assume then that people therefore believe what others tell them? Because if not, then there is no level of exaggeration that can scare them into quitting their bad habits and so no need for a third party whatsoever.

---------------------------------------------------------------------------------------------------------------------------
The second approach I missed while doing the test and only know thought of just now is to diagram seeing as how we do have a conditional statement in the conclusion which gives us: easily adopt--> doctor or other outsider provides warning. And so we need to show that the necc condition is truly necessary.

And so choice D shows that if the sufficient condition occurs (easily adopt) then we can have the necc condition not occur by saying that instead of listening to outsiders people can just decieve themselves. Is this hwhy D is right?

But still, why was A bad given my thoughts in the first part?

Let me know what you guys think=)

And sorry for all the posts, I am just always down to two choices it seems without really understanding how to knock the other chocie down. =/
 Emily Haney-Caron
PowerScore Staff
  • PowerScore Staff
  • Posts: 577
  • Joined: Jan 12, 2012
|
#21784
Hi Kristina,

This is a question where a pre-phrase will do wonders for you. Once you read the stimulus, you should be asking, "Why can't individuals easily use deception on their own?" The answer to that question - that individuals can't deceive themselves - is going to be your pre-phrase here. You can then use the assumption-negation technique to confirm you have the correct answer choice.

Whether people tend to believe whatever doctors tell them is irrelevant here; all we care about is whether people believe this particular piece of information (or other similar exaggerated pieces of information). So even if people do not tend to believe whatever doctors tell them, clearly they believe this piece of info about smoking, and perhaps also similar pieces of info (we don't know what they do believe and what they don't believe). Therefore, D is the much better answer.

Does that help?
 kristinaroz93
  • Posts: 160
  • Joined: Jul 09, 2015
|
#21789
I do get what you are saying! But was my 2nd approach incorrect for D in showing the necc conditon as being in fact necc correct?
 kristinaroz93
  • Posts: 160
  • Joined: Jul 09, 2015
|
#21800
? =)
 Clay Cooper
PowerScore Staff
  • PowerScore Staff
  • Posts: 241
  • Joined: Jul 03, 2015
|
#21807
Hi Kristina,

I think your second approach to D (if I understood it) was correct. Let me recap what I think you did and you can verify if I have understood it clearly.

You diagrammed from the stimulus: Indiv Easily Adopt --> Doctor Warned. Therefore, the negated for of D (which would state that individuals can in fact easily deceive themselves) would attack this conclusion by making the necessary condition of the prior rule in fact unnecessary. That is correct, and a useful and practical way to think about the question - it is a great example of why we stress the negation technique so much.

Thanks for your follow up, hopefully this will clarify the situation.
 kristinaroz93
  • Posts: 160
  • Joined: Jul 09, 2015
|
#21821
Hi clay,

Thanks so much!

Yes, I think you uderstood what I said : If people can easily decieve themselves, then you could easily adopt but without needing a third party to decieve you and thus the necc condition isn't really necc in the stimulus conclusion.


=)
 Johnclem
  • Posts: 122
  • Joined: Dec 31, 2015
|
#29807
Hello.,
Can someone please help me negate answer choice C . Here is my attempt: and I felt when negated it weakens the argument. And my Only reason for not choosing it was because I saw this choice as attacking the premise instead of the conclusion. :-?

the more the relevant danger is exaggerated , it is NOT more likely one is to break ones habit.


Thanks
John
 Emily Haney-Caron
PowerScore Staff
  • PowerScore Staff
  • Posts: 577
  • Joined: Jan 12, 2012
|
#29902
Hi John,

Yep, you've got it! Good job with the negation, and with identifying that this answer is not the correct answer because it weakens the wrong thing; in fact, it outright contradicts the premise.
 AnnBar
  • Posts: 33
  • Joined: Mar 24, 2017
|
#35750
Hello,

Could someone explain why answer B is incorrect? I chose D but wanted to get an idea why B was indeed incorrect. It seems like B does not deal with the conclusion directly.

Thank you in advance
User avatar
 Jonathan Evans
PowerScore Staff
  • PowerScore Staff
  • Posts: 727
  • Joined: Jun 09, 2016
|
#35816
Hi, AnnBar,

Good question! Let's revisit briefly what we need to do for an Assumption question such as this. The core task is to identify a necessary but unstated belief the author must have for his argument to be valid. In other words, the credited response will be a statement the truth of which is essential for the conclusion to work.

In this case, the author's conclusion is that people can't adopt this stop-smoking strategy unless some third-party provides the warning because (premise) the strategy involves deception.

Thus, we need to make a link between this premise involving deception and the conclusion insofar as it pertains to the warning coming from a third party. The most salient assumption will make a link between these concepts, as in the correct answer, (D), which you chose.

However, let's take a look at (B). Ask yourself, does the author really have to believe that most of the techniques for smoking work for other stuff? Is it possible that most of the techniques for smoking don't work for other stuff and for it still to be possible that people can't use this stop-smoking technique unless a third party is involved? The answer is yes: it is still possible that the author's conclusion could work with or without the truth of the statement in answer choice (B). This is the Assumption Negation Test™ in a nutshell.

I hope this helps!

Get the most out of your LSAT Prep Plus subscription.

Analyze and track your performance with our Testing and Analytics Package.